LSAT and Law School Admissions Forum

Get expert LSAT preparation and law school admissions advice from PowerScore Test Preparation.

User avatar
 Dave Killoran
PowerScore Staff
  • PowerScore Staff
  • Posts: 5853
  • Joined: Mar 25, 2011
|
#46266
Complete Question Explanation
(The complete setup for this game can be found here: lsat/viewtopic.php?t=14920)

The correct answer choice is (B)

If M is on team 2, then from the third rule neither R nor S can be on team 2:
O97_Game_#3_#18_diagram 1.png
Accordingly, answer choices (D) and (E) can be eliminated.

Because R cannot be on team 2, via the contrapositive of the fourth rule it follows that J cannot be on team 1. Because M is on team 2 and J cannot be on team 1, it follows that F must be on team 1:
O97_Game_#3_#18_diagram 2.png
Accordingly, answer choice (A) can be eliminated.

Because F is on team 1, from the first rule S cannot be on team 1. Because S cannot be on team 1 or team 2, and is thus sidelined for this question, we can infer that every other researcher must be included on a team. Accordingly, since J cannot be on team 1, J must be on team 2 and answer choice (B) is proven correct.
You do not have the required permissions to view the files attached to this post.
 Echx73
  • Posts: 36
  • Joined: Nov 11, 2015
|
#21390
Team Powerscore,

I am going through LSAT Game Type II and I found this question located on page 55. This is a Grouping: Defined-Fixed, Unbalanced: Overloaded Game.

On Question #18 it states, "If Marquez on Team 2, which on of the following must be on team 2? I know Jones cannot be on team 1 because it break the rules for R being on team 2. My question is, how do I prove Jones is in group two and not the extra researcher?

Thank you!

Eric
 David Boyle
PowerScore Staff
  • PowerScore Staff
  • Posts: 836
  • Joined: Jun 07, 2013
|
#21395
Echx73 wrote:Team Powerscore,

I am going through LSAT Game Type II and I found this question located on page 55. This is a Grouping: Defined-Fixed, Unbalanced: Overloaded Game.

On Question #18 it states, "If Marquez on Team 2, which on of the following must be on team 2? I know Jones cannot be on team 1 because it break the rules for R being on team 2. My question is, how do I prove Jones is in group two and not the extra researcher?

Thank you!

Eric
Hello Echx73,

The way you know is that Jones (and all other researchers besides Samuels) must be somewhere on the teams, is that there are 2 three-person teams--so that there must be 6 researchers--but S can't be anywhere, and has to be left out, since S can't be on team 1 (since F has to be on team 1, and F hates S), and can't be on team 2 either (since S hates M too). So J has to be somewhere...and it can't be on team 1, because if it were, then R would be on team 2, which is impossible since M hates R. So J must be on team 2.

Hope this helps,
David
 brebre234
  • Posts: 12
  • Joined: May 21, 2019
|
#77509
I know this question was done a while back but how is C wrong? In the rules it says each team must include at least one Anthropologist and 1 linguist. J is the Anthropologist while c is the linguist. I'm confused??
 Paul Marsh
PowerScore Staff
  • PowerScore Staff
  • Posts: 290
  • Joined: Oct 15, 2019
|
#77871
Hi brebre234! I think the explanation for this one is laid out pretty well above, but I'll run through it quickly again in case another perspective helps.

Our new rule tells us that M goes on team 2. From Rule 3 of our game, we know that since team 2 has M, that means team 2 can't have R or S. From the contrapositive of Rule 4, we know that since team 2 doesn't have R, that means team 1 doesn't have J. Since our setup tells us that each team needs at least one anthropologist and at least one linguist, we know we need an anthropologist in team 1, but M is already taken on team 2 and we know now that team 1 can't have J, so that leaves only F. So F has to go on team 1.

So since F is on team 1, from Rule 1 of our game we also know that S isn't on Team 1. But we also already figured out earlier that S can't go on Team 2. So that means that S isn't on either team. That means all the other 6 variables all need to go on either Team 1 or Team 2 (since there's only room for one variable to go on neither team). We already have M on Team 2 and F on Team 1. So the other 4 variables we need to place are J, N, O, and R. We figured out that R can't go in Team 2, so it has to go on Team 1. So since R is on Team 1, because of Rule 2 of our game, we know that N has to go on Team 2. And we figured out earlier that J can't go in Team 1, so it has to go on Team 2. So Team 2 is totally full: it contains M, N, and J. Which means Team 1 has to contain F, R, and O. And S is on neither team. So we have totally figured out where every variable needs to go!

Our question stem asks us which variable goes on Team 2, so we're looking for either N or J. (B) says J, so that's our right answer. We can definitively cross out (C) because we figured out that O has to go on Team 1.

Hope that helps! Please follow up if there's anything specific about this one that's not making sense.

Get the most out of your LSAT Prep Plus subscription.

Analyze and track your performance with our Testing and Analytics Package.